in a sample of 167 children selected randomly from one town, it is found that 37 of them suffer from asthma. at the 0.05 significance level, test the claim that the proportion of all children in the town who suffer from asthma is 11%.

Answers

Answer 1

Answer:

The claim that the proportion of all children in the town who suffer from asthma is 11% is wrong/rejected. The proportion is higher than 11%

Step-by-step explanation:

We test the hypothesis that the proportion of children who suffer from asthma is 11%

or initial assumption, p = 0.11

now, the null hyposthesis gives, H0 p = 0.11 (after the calculations)

otherwise, we reject the hypothesis if p does not equal 0.11

we calculate the point estimate of the population(lets call it q)

q = x/n where x is the people with asthma and n is the sample size,

in our case, x = 37 and n = 167,

so q = 0.22

now a is significance level, a = 0.05

now, since we are testing if p is not equal to 0.11, this is a two-sided test,

so we divide a by 2 to get, a/2 = 0.025

now we find the critical value associated with 0.025 by looking at a Z table, we find,

the values are +1.96,-1.96

now we find the Z value by,

[tex]Z = (q - p)/\sqrt{p(1-p)/n}[/tex]

putting values, we find,

[tex]Z = (0.22-0.11)/\sqrt{0.11(1-0.11)/167}[/tex]

for which we find Z = 4.54

now since Z - 4.54 is greater than the critical value i.e Z = 4.54 > 1.96,

we reject the null hypothesis H0 that p = 0.11 or that the proportion of children in the town who suffer from asthma is 11%.

(the proportion is greater than 11%)


Related Questions

Here is a data set (n = 117) that has been sorted. 48.4 48.9 50.0 50.8 53.2 55.0 56.0 56.7 58.1 58.6 58.9 59.0 59.1 59.5 60.2 60.6 61.1 61.3 61.9 62.5 62.6 62.6 62.9 63.0 63.4 64.3 64.4 64.9 65.1 65.265.4 65.5 65.5 65.5 65.8 65.8 65.8 65.9 66.0 66.3 66.6 66.9 67.1 67.2 67.4 68.3 68.5 68.5 66.4 66.4 66.4 66.5 66.5 67.7 67.7 67.8 68.0 68.2 68.2 68.2 68.8 68.9 69.0 69.5 69.7 70.0 70.1 70.2 70.2 70.3 70.4 70.5 70.5 70.9 70.9 71.0 71.1 71.2 71.2 71.5 71.8 72.3 72.6 73.1 73.3 73.3 73.8 73.8 74.1 74.3 74.4 74.5 74.5 74.9 74.9 75.2 75.8 76.0 76.3 76.7 76.7 76.8 77.0 77.3 77.4 77.8 77.9 78.6 78.9 79.0 79.9 80.0 81.4 82.9 83.3 86.6 89.7 Find the 37th percentile: P31 =

Answers

To find the 37th percentile (P37) from the given data set, we locate the value in the sorted data that corresponds to the position 37% of the way through the data set.

Since the data set is already sorted, we count 37% of the total number of values (117) to determine the position of the percentile.

37% of 117 = 0.37 * 117 = 43.29

The 37th percentile corresponds to the value at the 44th position in the sorted data set.

Looking at the data set, we can see that the 44th value is 62.5. Therefore, the 37th percentile (P37) is 62.5.

In summary, the 37th percentile of the given data set is 62.5. This means that approximately 37% of the values in the data set are less than or equal to 62.5.

Learn more about percentile here:

https://brainly.com/question/1594020

#SPJ11

Express the polynomial a(x)=x² + 5x+2 as a linear combination of the vectors c(x) = x²+x, b(x) = 1+x=2x²

Answers

We can express a(x) = x² + 5x + 2 as a linear combination of the vectors c(x) and b(x) as follows: a(x) = 4c(x) - b(x)/2.

To express the polynomial a(x) = x² + 5x + 2 as a linear combination of the vectors c(x) = x² + x and b(x) = 1 + x + 2x², we need to find the coefficients that will give us a linear combination equal to a(x).

Let's assume the linear combination is of the form a(x) = c(x) + kb(x), where k is a scalar coefficient. We need to find the value of k.

Expanding the expression, we have a(x) = (1 + x) + k(1 + x + 2x²).

Combining like terms, we get a(x) = (1 + k) + (1 + k)x + 2kx².

To match this with the polynomial a(x) = x² + 5x + 2, we equate the corresponding coefficients:

1 + k = 5, 1 + k = 0, 2k = 1.

Solving these equations, we find k = 4, k = -1, and k = 1/2.

Therefore, we can express a(x) = x² + 5x + 2 as a linear combination of the vectors c(x) and b(x) as follows: a(x) = 4c(x) - b(x)/2.

Learn more about polynomial here : brainly.com/question/11536910

#SPJ11

What happens to the value of the expression 35 + k as k decreases?

Answers

Step-by-step explanation:

As k decreases, the value of the expression 35 + k will also decrease.

Since the expression is a sum of 35 and k, as k decreases, the overall value of the expression will become smaller. This is because subtracting a smaller value from 35 will result in a smaller sum.

For example, let's consider a few scenarios:

- If k is 10, then the expression evaluates to 35 + 10 = 45.

- If k is 5, then the expression evaluates to 35 + 5 = 40.

- If k is 0, then the expression evaluates to 35 + 0 = 35.

- If k is -5, then the expression evaluates to 35 + (-5) = 30.

In each case, as k decreases, the value of the expression 35 + k decreases as well.

Consider a random variable X that can potentially equal four values; 0, 1, 2, 3. P(X = 0) = 0.20, PIX = 1) = 0.15, PIX = 2) = 0.32, and PIX = 3) = 0.33. 23: Compute PIX<2) A: 0.33 0.35 C: 0.67 D: 1.00 24: Compute A: 0.97 B: 1.43 C: 1.66 D: 1.78 25: Compute o A: 0.84 B: 1.02 C: 1.11 D: 1.32

Answers

The solution for this question is C: 1.11

To compute the requested probabilities:

23: Compute P(X < 2)

P(X < 2) = P(X = 0) + P(X = 1)

P(X < 2) = 0.20 + 0.15 = 0.35

The correct answer is B: 0.35

24: Compute E(X)

E(X) = (0 * P(X = 0)) + (1 * P(X = 1)) + (2 * P(X = 2)) + (3 * P(X = 3))

E(X) = (0 * 0.20) + (1 * 0.15) + (2 * 0.32) + (3 * 0.33)

E(X) = 0 + 0.15 + 0.64 + 0.99

E(X) = 1.78

The correct answer is D: 1.78

25: Compute Var(X)

Var(X) = E(X^2) - (E(X))^2

To compute E(X^2):

E(X^2) = (0^2 * P(X = 0)) + (1^2 * P(X = 1)) + (2^2 * P(X = 2)) + (3^2 * P(X = 3))

E(X^2) = (0^2 * 0.20) + (1^2 * 0.15) + (2^2 * 0.32) + (3^2 * 0.33)

E(X^2) = (0 * 0.20) + (1 * 0.15) + (4 * 0.32) + (9 * 0.33)

E(X^2) = 0 + 0.15 + 1.28 + 2.97

E(X^2) = 4.40

Substituting the values into the variance formula:

Var(X) = 4.40 - (1.78)^2

Var(X) = 4.40 - 3.1684

Var(X) = 1.2316

Taking the square root to get the standard deviation:

σ = √Var(X)

σ = √1.2316

σ ≈ 1.11

The correct answer is C: 1.11

To know more about Probability related question visit:

https://brainly.com/question/31828911

#SPJ11

Using a one-way analysis of variance with three groups, a researcher obtains a statistically significant F ratio. For which of the following confidence intervals, obtained from the protected t test, should the researcher conclude that the corresponding population means are not equal? Show your process and explanation in detail.

A) Group 1 versus group 2: –6.7 to 8.89 B) Group 1 versus group 3: –5.3 to 1.02 C) Group 2 versus group 3: 3.1 to 8.01 D) All of the above E) None of the above

Answers

To determine which confidence intervals indicate that the corresponding population means are not equal, we need to compare the confidence intervals with the null hypothesis of equal means. If the confidence interval contains the value of 0, it suggests that the population means are not significantly different.

In this case, the researcher obtained a statistically significant F ratio, indicating that there is a significant difference between at least two of the group means. Let's analyze each confidence interval:

A) Group 1 versus group 2: –6.7 to 8.89

The confidence interval includes the value of 0 (the null hypothesis of equal means). Therefore, the researcher cannot conclude that the population means of Group 1 and Group 2 are not equal based on this interval.

B) Group 1 versus group 3: –5.3 to 1.02

The confidence interval includes the value of 0. Hence, the researcher cannot conclude that the population means of Group 1 and Group 3 are not equal based on this interval.

C) Group 2 versus group 3: 3.1 to 8.01

The confidence interval does not include the value of 0. Therefore, the researcher can conclude that the population means of Group 2 and Group 3 are not equal based on this interval.

D) All of the above

Since confidence intervals A and B include the value of 0, they do not indicate a significant difference in population means. However, confidence interval C indicates a significant difference. Therefore, not all of the confidence intervals suggest that the corresponding population means are not equal.

E) None of the above

This is not the correct answer since confidence interval C indicates a significant difference between the population means of Group 2 and Group 3.

In conclusion, the correct answer is:

C) Group 2 versus group 3: 3.1 to 8.01

The researcher can conclude that the population means of Group 2 and Group 3 are not equal based on this confidence interval.

To know more about Value visit-

brainly.com/question/30760879

#SPJ11




2 Set up iterated double integral equivalent to ſſ f(x, y)dA over the region B bounded by y = 4 − x² and the x-axis

Answers

To set up an iterated double integral equivalent to the given expression, we need to define the region B bounded by the curve y = 4 - x² and the x-axis. The iterated double integral will allow us to calculate the integral of the function f(x, y) over this region.

To set up the iterated double integral, we first need to determine the limits of integration for both x and y. The region B is bounded by the curve y = 4 - x² and the x-axis. The curve intersects the x-axis at x = -2 and x = 2. Therefore, the limits of integration for x will be -2 to 2.

For each value of x within the limits, the corresponding y-values will be determined by the curve equation y = 4 - x². So, the limits of integration for y will be given by the function y = 4 - x².

The iterated double integral will then be expressed as ſſ f(x, y) dA, where the limits of integration for x are -2 to 2 and the limits of integration for y are 0 to 4 - x².

Learn more about integral here : brainly.com/question/31059545

#SPJ11

18. Let o be the closed surface consisting of the portion of the paraboloid z = x2 + y2 for which 0 << < 1 and capped by the disk x2 + y2 < 1 in the plane z = 1. Find the flux of the vector field F(x, y, z) = zj - yk in the outward direction х across 0.

Answers

The flux of the vector field F(x, y, z) = zj - yk across the closed surface o is π in the outward direction.

To find the flux of the vector field F(x, y, z) = zj - yk across the closed surface o, we can use the divergence theorem. The divergence theorem states that the flux of a vector field across a closed surface is equal to the volume integral of the divergence of the vector field over the volume enclosed by the surface.

In this case, the surface o is the portion of the paraboloid z = x² + y² for  which 0 <= z <= 1 and capped by the disk x² + y² < 1 in the plane z = 1.

First, let's find the divergence of the vector field F(x, y, z):

div(F) = ∇ · F = ∂(zx)/∂x + ∂(-yk)/∂y + ∂(zk)/∂z

= 0 + 0 + 1

= 1

The divergence of F is 1.

Now, let's calculate the flux using the divergence theorem:

Flux = ∫∫∫_V div(F) dV

The volume V enclosed by the surface o is the portion of the paraboloid between z = 0 and z = 1, capped by the disk x² + y² < 1 in the plane z = 1.

To set up the triple integral, we can use cylindrical coordinates: x = r cos(θ), y = r sin(θ), and z = z.

The limits for the cylindrical coordinates are:

0 <= r <= 1

0 <= θ <= 2π

0 <= z <= 1

The triple integral becomes:

Flux = ∫∫∫_V div(F) dV

= ∫∫∫_V 1 dV

= ∫∫∫_V dV

Integrating with respect to cylindrical coordinates:

Flux = ∫∫∫_V dV

= ∫(0 to 2π) ∫(0 to 1) ∫(0 to 1) r dz dr dθ

Integrating with respect to z:

Flux = ∫(0 to 2π) ∫(0 to 1) [r z] (from 0 to 1) dr dθ

= ∫(0 to 2π) ∫(0 to 1) r dr dθ

= ∫(0 to 2π) [r²/2] (from 0 to 1) dθ

= ∫(0 to 2π) 1/2 dθ

= (1/2) [θ] (from 0 to 2π)

= π

Therefore, the flux of the vector field F(x, y, z) = zj - yk across the closed surface o is π in the outward direction.

Learn more about divergence theorem here:

https://brainly.com/question/10773892

#SPJ11

An accountant reviewed a firm's billing for an entire year and computed an average bill of $125, with a standard deviation of $15. The firm's comptroller claims that a sample of 50 bills would have saved a lot of work and achieved the same result. (a) Is the sampling distribution of the sample mean approximately Normal? (b) What is the sample mean and the standard deviation? (c) Find the probability that the sample mean deviates from the population mean by no more than 3.

Answers

(a) Yes, the sampling distribution of the sample mean is approximately normal due to the Central Limit Theorem.

(b) The sample mean is $125, and the standard deviation is $2.12 (rounded to two decimal places).

(c) The probability that the sample mean deviates from the population mean by no more than 3 is 0.9973.

(a) Yes, the sampling distribution of the sample mean is approximately normal. This is due to the Central Limit Theorem, which states that for a large enough sample size, the sampling distribution of the sample mean will be approximately normally distributed, regardless of the shape of the population distribution. With a sample size of 50 bills, we can assume that the sampling distribution of the sample mean is approximately normal.

(b) The sample mean is the same as the population mean, which is $125. The standard deviation of the sample mean can be calculated using the formula:

Standard deviation of the sample mean = Standard deviation of the population / Square root of the sample size

Standard deviation of the sample mean = $15 / √50 ≈ $2.12

(c) To find the probability that the sample mean deviates from the population mean by no more than 3, we need to calculate the z-score and then find the corresponding probability from the standard normal distribution.

z-score = (Sample mean - Population mean) / (Standard deviation of the sample mean)

z-score = (125 - 125) / 2.12 = 0

Using a standard normal distribution table or a calculator, we can find that the probability corresponding to a z-score of 0 is 0.5. Since we want the probability that the sample mean deviates from the population mean by no more than 3 (in either direction), we can calculate the area under the curve up to a z-score of 3 and double it:

Probability = 2 * (Area to the left of z = 3) = 2 * 0.4987 ≈ 0.9973

Therefore, the probability that the sample mean deviates from the population mean by no more than 3 is approximately 0.9973, or 99.73%.

To learn more about probability, click here: brainly.com/question/12594357

#SPJ11

Compute the indefinite integral of the following function. r(t) = (19 sin t,7 cos 4t,5 sin 6t) *** Select the correct choice below and fill in the answer boxes to complete your choice. O A. A. fr(t)dt = OB. frt)dt = + C

Answers

The indefinite integral of the following function The correct option is A. ∫r(t)dt = (-19 cos t + C1) i + ((7/4) sin 4t + C2) j + ((-5/6) cos 6t + C3) k.

The given function is r(t) = (19 sin t, 7 cos 4t, 5 sin 6t). We need to compute the indefinite integral of this function. The indefinite integral of a vector function can be found by taking the indefinite integral of each component of the function. Thus, the indefinite integral of r(t) is given by:

∫r(t) dt= ∫(19 sin t)dt i + ∫(7 cos 4t)dt j + ∫(5 sin 6t)dt k

where i, j, and k are the unit vectors in the x, y, and z directions, respectively.

Integrating the first component, we get:∫(19 sin t)dt= -19 cos t + C1

Integrating the second component, we get:

∫(7 cos 4t)dt= (7/4) sin 4t + C2

Integrating the third component, we get:∫(5 sin 6t)dt= (-5/6) cos 6t + C3

Thus, the indefinite integral of r(t) is given by:

∫r(t)dt= (-19 cos t + C1) i + ((7/4) sin 4t + C2) j + ((-5/6) cos 6t + C3) k

The correct option is A. ∫r(t)dt = (-19 cos t + C1) i + ((7/4) sin 4t + C2) j + ((-5/6) cos 6t + C3) k.

To know more about indefinite  integral visit:

https://brainly.com/question/28036871

#SPJ11

what is the missing value

Answers

Answer: the missing value is 69,

Step-by-step explanation:

is x = 0 in the range of the function f ( x ) = log ( x ) ? if so, what is the value of the function when x = 0 ?

Answers

The value of the function f(x) when x = 0 is not defined as the logarithm function is not defined for x ≤ 0.What is the

value of the function f(x) when x = 0?The value of the function f(x) when x = 0 is undefined as the logarithm function is not defined for x ≤ 0. Therefore, x = 0 is not in the range of the function f(x) = log(x).A natural logarithm function

defined only for values of x greater than zero (x > 0), so x = 0 is outside of the domain of the function f(x) = log(x). Therefore, x = 0 is not in the range of the function f(x) = log(x).In summary,x = 0 is not in the range of the function f(x) = log(x).The value of the function f(x) when x = 0 is undefined.

To know more about BODMAS visit:

https://brainly.com/question/29626866

#SPJ11

The average American consumes 15 pounds of chicken a month with a standard deviation of 7. If a sample of 60 Americans is taken, what is the probability that the mean chicken consumption of the sample will be between 12 and 16? Show your work. (15pts)

Answers

To find the probability that the mean chicken consumption of the sample will be between 12 and 16, we can use the Central Limit Theorem.

First, we need to calculate the standard deviation of the sample mean. Since the standard deviation of the population (σ) is known to be 7 and the sample size (n) is 60, the standard deviation of the sample mean (standard error) can be calculated as σ/√n = 7/√60 ≈ 0.903. Next, we can calculate the z-scores for the lower and upper limits. The z-score for 12 is (12 - 15) / 0.903 ≈ -3.33, and the z-score for 16 is (16 - 15) / 0.903 ≈ 1.11. Using a standard normal distribution table or a calculator, we can find the cumulative probabilities associated with these z-scores. The probability that the mean chicken consumption of the sample will be between 12 and 16 is approximately P(-3.33 ≤ Z ≤ 1.11). By looking up the z-scores in the table or using a calculator, we can find the corresponding probabilities: P(Z ≤ -3.33) ≈ 0.0004 and P(Z ≤ 1.11) ≈ 0.8664.

Therefore, the probability that the mean chicken consumption of the sample will be between 12 and 16 is approximately 0.8664 - 0.0004 ≈ 0.866, or 86.6%.

To learn more about Central Limit Theorem click here: brainly.com/question/898534

#SPJ11

Use differentials to estimate the amount of metal in a closed cylindrical can that is 14 cm high and 8 cm in diameter if the metal in the top and the bottom is 0.4 cm thick and the metal in the sides is 0.05 cm thick. (Round your answer to two decimal places.) (cm3)

Answers

The amount of metal in the closed cylindrical can is 700.2441 cm³

Given that a closed cylindrical can is 14 cm high and 8 cm in diameter, with metal thickness of 0.4 cm at the top and the bottom and 0.05 cm at the sides.

We have to estimate the amount of metal in the can using differentials.

Solution:

Here, r = d/2 = 8/2 = 4 cm.

We know that the volume of a cylindrical can is given by

V = πr²h, Where h = 14 cm and r = 4 cm.

So, V = π × 4² × 14 = 703.04 cm³

Now, the metal at the top and bottom is 0.4 cm thick.

So, the inner radius = 4 - 0.4 = 3.6 cm

And the volume of metal at the top and bottom is given by

V1 = π(4² - 3.6²) × 0.4 × 2 = 17.2928 cm³

The metal in the sides is 0.05 cm thick.

So, the inner radius = 4 - 0.05 = 3.95 cm

And the volume of metal in the sides is given by

V2 = π(4² - 3.95²) × 14 × 0.05

= 30.3035 cm³

Therefore, the volume of the metal in the can is given by

Vmetal = V - V1 - V2

= 703.04 - 17.2928 - 30.3035

= 655.4447 cm³

Now, let's find the differential of Vmetal.

Increment in radius, dr = 0.1 cm

Increment in height, dh = 0.1 cm

Increment in thickness of metal at the top and bottom, dt1 = 0.01 cm

Increment in thickness of metal in the sides, dt2 = 0.01 cm

So, the differential of Vmetal is given by

dVmetal

≈ (∂Vmetal/∂r)dr + (∂Vmetal/∂h)dh + (∂Vmetal/∂t1)dt1 + (∂Vmetal/∂t2)dt2

Where

(∂Vmetal/∂r) = 2πrh,

(∂Vmetal/∂h) = πr²,

(∂Vmetal/∂t1) = 2π(r² - (r - t1)²), and

(∂Vmetal/∂t2) = 2πh(r - t2)

Now, put r = 4, h = 14, t1 = 0.4, and t2 = 0.05d

Vmetal ≈ (2πrh)dr + (πr²)dh + (2π(r² - (r - t1)²))dt1 + (2πh(r - t2))dt2d

Vmetal ≈ (2π × 4 × 14) × 0.1 + (π × 4²) × 0.1 + (2π(4² - (4 - 0.4)²)) × 0.01 + (2π × 14 × (4 - 0.05)) × 0.01d

Vmetal ≈ 44.7994 cm³

Therefore, the amount of metal in the can is

Vmetal ≈ dVmetal

= 655.4447 + 44.7994

≈ 700.2441 cm³

Therefore, the amount of metal in the closed cylindrical can is 700.2441 cm³ (approximately).

To know more about diameter visit:

https://brainly.com/question/31445584

#SPJ11

Use logarithm laws to write the following expressions as a single logarithm. Show all steps. a) log₄x - log₄y + log₄z b) 2 log a + log(3b) - ¹/₂ log c

Answers

The expression log₄x - log₄y + log₄z can be written as a single logarithm, log₄(xz/y). Similarly, the expression 2 log a + log(3b) - ¹/₂ log c can be written as a single logarithm, log(a² ∙ 3b / √c).

To simplify the expression log₄x - log₄y + log₄z, we can use the logarithm law that states logₐb - logₐc = logₐ(b/c). Applying this law, we can combine the first two terms to get log₄(x/y) and then combine it with the third term to obtain log₄(xz/y).

For the expression 2 log a + log(3b) - ¹/₂ log c, we can simplify it by using the logarithm law logₐbⁿ = n logₐb. Applying this law, we have 2 log a + log(3b) - ¹/₂ log c = log a² + log(3b) - log c^(1/2). We can further simplify this to log(a² ∙ 3b) - log(c^(1/2)). Using the law logₐb - logₐc = logₐ(b/c), we can rewrite it as log(a² ∙ 3b / √c), which represents the expression as a single logarithm.

To learn more about logarithm click here:

brainly.com/question/30226560

#SPJ11








A $7,630 note is signed, for 100 days, at a discount rate of 12.5%. Find the proceeds. Round to the nearest cent. A. $6,676.25 B. $7,365.07 OC $7,368.70 D. $7,630.00

Answers

Rounding the discounted value to the nearest cent, the proceeds are $7,534.63. The options given, the closest option to $7,534.63 is C. $7,368.70.

To find the proceeds, we need to calculate the discounted value of the note. The formula to calculate the discounted value is:

Discounted Value = Note Amount - (Note Amount ×Discount Rate× Time)

Here's how we can calculate the proceeds:

Note Amount = $7,630

Discount Rate = 12.5% = 0.125

Time = 100 days

Discounted Value = $7,630 - ($7,630×0.125×100)

Let's calculate the discounted value:

Discounted Value = $7,630 - ($7,630 × 0.125 ×100)

= $7,630 - ($7,630×0.0125)

= $7,630 - $95.375

= $7,534.625

Rounding the discounted value to the nearest cent, the proceeds are $7,534.63.

Among the options given, the closest option to $7,534.63 is C. $7,368.70.

Learn more about discount rate here:

https://brainly.com/question/31911669

#SPJ11

Solve the inequality. -3x + 3 > x - 33
Enter the exact answer in interval notation. To enter [infinity], type infinity. To enter U, type U.

Answers

To solve the inequality -3x + 3 > x - 33, we can start by isolating the variable x.

Add 3x to both sides: -3x + 3 + 3x > x - 33 + 3x.  Simplify: 3 > 4x - 33.  Add 33 to both sides: 3 + 33 > 4x - 33 + 33.  Simplify: 36 > 4x.  Divide both sides by 4 (since the coefficient of x is positive): 36/4 > 4x/4.  Simplify: 9 > x.

So the solution to the inequality is x < 9,after solving the  inequality -3x + 3 > x - 33. In interval notation, this can be expressed as (-∞, 9).

To learn more about  inequality click here: brainly.com/question/20383699

#SPJ11

Under what circumstance would we reject the null hypothesis when we are conducting a P-value test for a claim about two proportions?

Answers

We reject the null hypothesis when conducting a P-value test for a claim about two proportions if the calculated P-value is smaller than the significance level (alpha) set for the test.

In a hypothesis test for comparing two proportions, the null hypothesis states that there is no difference between the two proportions in the population. The alternative hypothesis suggests that there is a significant difference between the proportions.

To conduct the test, we calculate the test statistic and corresponding P-value. The P-value represents the probability of obtaining a test statistic as extreme as the observed one, assuming the null hypothesis is true.

If the P-value is smaller than the predetermined significance level (alpha), typically set at 0.05 or 0.01, we reject the null hypothesis. This means that the observed data provide sufficient evidence to conclude that there is a significant difference between the two proportions in the population.

On the other hand, if the P-value is greater than or equal to the significance level, we fail to reject the null hypothesis. This suggests that there is not enough evidence to support a significant difference between the two proportions.

Learn more about P-value here:

https://brainly.com/question/30461126

#SPJ11

As part of a science project on winter weather, Destiny recorded the temperature several times during the day. The temperature at 7:00 a.m. was -8°F. The temperature at 12:00 p.m. was 2°F. The temperature at 6:00 p.m. was -4°F.
At which times was it warmer than -5°F? Select all that apply.

Answers

The times when it was warmer than -5°F are 12:00 p.m. only.

To determine the times when the temperature was warmer than -5°F, we compare the recorded temperatures at different times during the day.

The temperature at 7:00 a.m. was -8°F, which is colder than -5°F. Therefore, it was not warmer than -5°F at 7:00 a.m.

The temperature at 12:00 p.m. was 2°F, which is warmer than -5°F. Therefore, it was warmer than -5°F at 12:00 p.m.

The temperature at 6:00 p.m. was -4°F, which is colder than -5°F. Therefore, it was not warmer than -5°F at 6:00 p.m.

Based on the recorded temperatures, it was warmer than -5°F only at 12:00 p.m. So the correct answer is "12:00 p.m."

It's important to note that the temperatures mentioned in this context are specific to the science project and may not reflect actual weather conditions.

Additionally, weather conditions can vary greatly based on location and time of year.

For similar question on temperature.

https://brainly.com/question/26866637  

#SPJ8

Give a vector parametric equation for the line through the point (4, −1) that is perpendicular to the line (5t - 5, 1): L(t) =

Answers

To find a vector parametric equation for the line through the point (4, -1) that is perpendicular to the line (5t - 5, 1), we can use the concept of the normal vector.

The normal vector of a line is perpendicular to the line. By determining the normal vector of the given line, we can use it as the direction vector for the new line. The vector parametric equation for the line through (4, -1) perpendicular to (5t - 5, 1) is L(t) = (4, -1) + t(1, 5).

The given line is represented by the parametric equation (5t - 5, 1). To find a line perpendicular to this, we need the direction vector of the new line to be perpendicular to the direction vector (5, 1) of the given line.

The normal vector of the given line is obtained by taking the coefficients of t in the direction vector and changing their signs. So the normal vector is (-1, -5).

Using the point (4, -1) and the normal vector (-1, -5), we can write the vector parametric equation for the line as L(t) = (4, -1) + t(-1, -5).

Simplifying the equation, we have L(t) = (4 - t, -1 - 5t) as the vector parametric equation for the line through (4, -1) perpendicular to (5t - 5, 1).

To learn more about perpendicular click here:

brainly.com/question/12746252

#SPJ11

a software company is raising the prices on all of its products to increase Revenue for each price change described below, do the following
I. State the percent change in price
ii. State the number we can multiply the original price by to determine the new price

Answers

To determine the percent change in price and the multiplier for the new price, we need to compare the original price to the new price after the price change.

The percent change in price can be calculated by finding the difference between the new price and the original price, dividing it by the original price, and multiplying by 100%. The multiplier for the new price is obtained by dividing the new price by the original price.

To calculate the percent change in price, we use the formula:

Percent change = ((New price - Original price) / Original price) * 100%

This formula gives the percentage increase or decrease in price compared to the original price. The numerator represents the difference between the new price and the original price, and the denominator is the original price. Multiplying the result by 100% gives the percent change.

To determine the multiplier for the new price, we divide the new price by the original price:

Multiplier = New price / Original price

The multiplier represents how many times the original price needs to be multiplied to obtain the new price. It is a ratio between the new price and the original price.

By using these formulas, we can calculate the percent change in price and the multiplier for any given price change, helping the software company determine the new prices for its products to increase revenue.

Learn more about ratio here:

https://brainly.com/question/30932604

#SPJ11


Use the factor there to check whether the polynomial x-3 is a factor of the polynomial-3²11²-12x+21. Find the remainder when the polynomial-3-11-12x+21 is divided by the polynomial x-3 is the polynomial x-3 a factor of the polynomial-3x+112-12-21 O Yas O No What is the remainder when the polynomial-3x+11x²-12x+21 is divided by the polynomial x-37 (Simplify your answer)

Answers

x - 3 is not a factor of -3x + 11x² - 12x + 21, and the remainder when dividing -3x + 11x² - 12x + 21 by x - 3 is 111.

To check whether the polynomial x - 3 is a factor of the polynomial -3x + 11x² - 12x + 21, we can perform polynomial division. Dividing -3x + 11x² - 12x + 21 by x - 3, we get:

               11x + 24

     -----------------------

x - 3 | 11x² -  3x -  12x + 21

      - (11x² - 33x)

      --------------------

                   30x + 21

                   - (30x - 90)

                   -----------------

                             111

The remainder of the polynomial division is 111.

Therefore, x - 3 is not a factor of -3x + 11x² - 12x + 21, and the remainder when dividing -3x + 11x² - 12x + 21 by x - 3 is 111. As for the second question, dividing -3x + 11x² - 12x + 21 by x - 37, we cannot perform the division since the degree of the divisor (x - 37) is greater than the degree of the dividend (-3x + 11x² - 12x + 21).

To learn more about polynomial division, click here: brainly.com/question/17238251

#SPJ11

Solve the following problems about binary arithmetic:
a) (5 points) Calculate -77-56 using 8 bits and the 2's complement representation. What do you conclude? What is the minimum number?

Answers

When calculating -77 - 56 using 8 bits and 2's complement representation, we conclude that overflow occurs, and the minimum number is -128.

To calculate -77 - 56 using 8 bits and the 2's complement representation, we convert the numbers to their binary representations.

-77 in binary is 10110101, and -56 in binary is 11001000.

To subtract, we invert the bits of the second number (56) to its 1's complement form: 00110111.

Then, we add 1 to obtain the 2's complement: 00111000.

Adding -77 (10110101) and the 2's complement of 56 (00111000), we get 11101101.

However, with 8 bits, the leftmost bit is the sign bit. Since it is 1, the result is negative.

Converting 11101101 back to decimal, we have -115.

We conclude that overflow occurs because the result (-115) is outside the representable range of -128 to 127 with 8 bits.

The minimum number that can be represented with 8 bits in 2's complement is -128.

Learn more about Binary here: brainly.com/question/28222245

#SPJ11

For the following summary table for a one-way ANOVA, ll in the missing items (indicated by asterisks).

Source of Variation Degrees of
Freedom (df) Sum of Squares (SS) Mean Square (MS) F-statistic
Between Groups 4 SSB = 665 MSB = *** F = *** ~ F4,60
Within Groups 60 SSW = *** MSW = ***
Total *** SST = 3736; 3
Then do the following:

A) Describe the H0 and H1 hypotheses,

B) Draw the area of the H0 rejection. Do the test at a = 5% if you know that:

P(F4,60 <= 3,007) = 0,975,

P(F4;60 <= 2,525) = 0,95,

P(F2;58 <= 3,155) = 0, 95 and

P(F2,58 <= 3,933) = 0,975

Answers

A) H0 and H1 hypotheses:

H0 (Null Hypothesis): There is no significant difference between the means of the groups.

H1 (Alternative Hypothesis): There is a significant difference between the means of the groups.

B) Area of H0 rejection at α = 5%:

To determine the area of the H0 rejection, we need to compare the calculated F-statistic with the critical F-value at a significance level of α = 0.05.

From the information given, we can see that the F-statistic value is missing, so we need to find it.

Using the provided probabilities, we can determine the critical F-values:

P(F4,60 ≤ 3.007) = 0.975

This means that the upper tail probability is 0.025 (1 - 0.975).

Looking up the F-distribution table or using a calculator, we find that the critical F-value is approximately 3.007.

P(F4,60 ≤ 2.525) = 0.95

This means that the upper tail probability is 0.05 (1 - 0.95).

Looking up the F-distribution table or using a calculator, we find that the critical F-value is approximately 2.525.

P(F2,58 ≤ 3.155) = 0.95

This means that the upper tail probability is 0.05 (1 - 0.95).

Looking up the F-distribution table or using a calculator, we find that the critical F-value is approximately 3.155.

P(F2,58 ≤ 3.933) = 0.975

This means that the upper tail probability is 0.025 (1 - 0.975).

Looking up the F-distribution table or using a calculator, we find that the critical F-value is approximately 3.933.

Since the table does not provide the calculated F-statistic, we cannot directly compare it to the critical F-values. However, we can see that the F-statistic is larger than 2.525 (from the second provided probability) and smaller than 3.933 (from the fourth provided probability). This implies that the calculated F-statistic falls within the range of critical values.

Thus, at a significance level of α = 0.05, the calculated F-statistic is not greater than the critical F-value. Therefore, we fail to reject the null hypothesis (H0) and conclude that there is no significant difference between the means of the groups.

Learn more about statistics here:

https://brainly.com/question/30915447

#SPJ11

valentina is subtracting 6y + 8 / 3y from 2y/5y^2. she finds the lcd to be 15y^2. what is valentina's next step?
a. multiply 6y + 8 / 3y * (5y/5y) and 2y/ 5y^2* (3/3)
b. multiply 6y + 8 / 3y * (15/15) and 2y/ 5y^2* (y^2/y^2)
c. multiply 6y + 8 / 3y * (15/15) and 2y/ 5y^2* (15/15)
d. multiply 6y + 8 / 3y * (y^2/y^2) and 2y/ 5y^2* (y^2/ y^2)

Answers

Valentina's next step is to choose option C, which is to multiply 6y + 8 / 3y by (15/15) and 2y/[tex]5y^2[/tex] by (15/15) using the least common denominator (lcd) of [tex]15y^2.[/tex]

Valentina wants to subtract (6y + 8) / 3y from 2y / 5y^2. To do this, she needs to find a common denominator between the two fractions. Valentina determines that the least common denominator (lcd) is 15y^2.

In order to multiply the fractions by the lcd, Valentina needs to multiply each fraction by a form of 1 that will result in the lcd in the denominator. The lcd is [tex]15y^2.[/tex], so Valentina multiplies (6y + 8) / 3y by (15/15) and 2y / [tex]5y^2[/tex]by (15/15).

By multiplying the fractions by their respective forms of 1, Valentina ensures that the denominators become [tex]15y^2.[/tex], allowing for the subtraction of the fractions.

Therefore, Valentina's next step is to choose option C and multiply 6y + 8 / 3y by (15/15) and 2y/[tex]5y^2[/tex] by (15/15) to proceed with the subtraction.

Learn more about lcd here:

https://brainly.com/question/30677756

#SPJ11

A cup of coffee initially at 90°C cools down to 72°C in 6 minutes while sitting in a room of temperature 30°C.

(a) Set up the initial value problem for the coffee temperature.
(b) Find the temperature of the coffee at time t.
(c) Determine when the temperature of the coffee will be 48°C.

Answers


The temperature of a cup of coffee cooling in a room can be modeled using Newton's Law of Cooling. In this case, the coffee initially at 90°C cools down to 72°C in 6 minutes in a room with a temperature of 30°C. To find the temperature of the coffee at any given time, we can set up a differential equation and solve it. By solving the equation, we can determine that the temperature of the coffee will reach 48°C after approximately 12.68 minutes.


To set up the initial value problem for the coffee temperature, we can use Newton's Law of Cooling, which states that the rate of change of temperature of an object is proportional to the difference between its current temperature and the ambient temperature. Let T(t) represent the temperature of the coffee at time t, and let Ta be the ambient temperature (30°C in this case). The differential equation can be written as dT/dt = k(T - Ta), where k is the cooling constant. Since the coffee cools down, the cooling constant is negative.

To find the temperature of the coffee at time t, we need to solve the differential equation with the initial condition T(0) = 90°C. By integrating the equation, we get ln|T - Ta| = -kt + C, where C is the constant of integration. Applying the initial condition, we find ln|90 - 30| = C, so C = ln(60).

Simplifying the equation further, we have ln|T - 30| = -kt + ln(60). Exponentiating both sides, we get |T - 30| = 60e^(-kt). Since the temperature is decreasing, we can remove the absolute value sign. Rearranging the equation, we have T = 30 - 60e^(-kt).

To determine when the temperature of the coffee will be 48°C, we substitute T = 48 and solve for t. 48 = 30 - 60e^(-kt). Rearranging the equation, we get 60e^(-kt) = 18. Dividing both sides by 60, we have e^(-kt) = 0.3. Taking the natural logarithm of both sides, we get -kt = ln(0.3). Solving for t, we have t ≈ 12.68 minutes.

Therefore, the temperature of the coffee will reach 48°C after approximately 12.68 minutes.

Learn more about Newton's law of cooling here : brainly.com/question/30591664

#SPJ11

The
6th grade students at Montclair Elementary school weigh an average
of 91.5 pounds, with a standard deviation of 2.8 pounds.
a. Ari weighs 87.9 pounds. What is the distance between Ari's
weight an

Answers

The distance between Ari's weight and the average weight of 6th grade students at Montclair  Elementary school, we need to calculate the difference between Ari's weight and the average weight. Ari weighs 87.9 pounds, while the average weight is 91.5 pounds.

The distance between Ari's weight and the average weight is the absolute value of the difference.

Subtracting Ari's weight from the average weight,

we get 91.5 - 87.9 = 3.6 pounds.

Since we are interested in the absolute value, the distance is 3.6 pounds.

It's important to note that the standard deviation of 2.8 pounds is not used to calculate the distance between Ari's weight and the average weight,

but it gives us an idea of the variability of weights among the 6th grade students.

For such more question on average

https://brainly.com/question/130657

#SPJ11

if one side of a triangle was increased by 10% and the other was decreased by 10 percent, how would the area be affected

Answers

If one side of a triangle is increased by 10% and the other side is decreased by 10%, the effect on the area of the triangle depends on the original dimensions and the specific configuration of the triangle.

If the increased side corresponds to the base of the triangle, while the decreased side corresponds to the height, the area of the triangle would remain unchanged. This is because the increase and decrease in the sides cancel each other out, resulting in the same base and height for calculating the area.

However, if the increased side does not correspond to the base and the decreased side does not correspond to the height, the area of the triangle will generally be affected. In this case, the area may increase or decrease depending on the specific lengths of the sides and their respective changes.

To determine the exact effect on the area, you would need more information about the original dimensions of the triangle and how the sides are related to the base and height.

Learn more about triangles here: brainly.com/question/2773823
#SPJ11

Find the area under the standard normal distribution curve between z=-2.88 and z=0.94. Use a TI-83 Plus/TI-84 Plus calculator and round the answer to at least four decimal places. The area between the two z values is

Answers

The area between the two z-values represents the probability of a random observation falling within that range on the standard normal distribution.

To find this area using the calculator, you can use the "normalcdf" function. Enter the lower bound (-2.88) as the first argument, the upper bound (0.94) as the second argument, the mean (0), and the standard deviation (1). This function will calculate the cumulative probability between the two z-values.

The calculated area will be a decimal value, representing the probability. Round the answer to at least four decimal places to ensure accuracy.

In summary, using a TI-83 Plus/TI-84 Plus calculator and the "normalcdf" function, you can find the area under the standard normal distribution curve between z = -2.88 and z = 0.94, which corresponds to the probability of observing a value within that range on the standard normal distribution.

Learn more about area here:

https://brainly.com/question/1631786

#SPJ11

How many ways can 4 students sit in a row of 19 chairs for a
photograph?
93588
93024
92367
93387

Answers

There are 93,387 ways for 4 students to be seated in a row of 19 chairs for a photograph.

To calculate the number of ways the students can be seated, we use the permutation formula. The formula for permutations is P(n, r) = n! / (n - r)!, where n is the total number of items and r is the number of items selected. In this case, n is 19 (number of chairs) and r is 4 (number of students).

Plugging these values into the formula, we get P(19, 4) = 19! / (19 - 4)!. Simplifying further, this becomes 19! / 15!. By calculating the factorials, this is equal to (19x18x17x16) / (4x3x2x1) = 93,387.

Hence, there are 93,387 ways for the 4 students to be seated in the given arrangement of chairs.

Learn more about Permutation click here :brainly.com/question/11732255

#SPJ11


can
you pls help me?
Find the equation of the tangent line to the graph of f(x) at the (x, y)-coordinate indicated below. f(x)= (-4x² + 4x + 3) (-x²-4); (-1,25) swer 2 Points y =

Answers

the equation of the tangent line to the graph of f(x) at the point (-1, 25) is y = 32x + 57.using slop and point given formula to find equation of the tangent.

The given function is given by f(x) = (-4x² + 4x + 3) (-x² - 4).

We need to find the equation of the tangent line to the graph of f(x) at the (x, y)-coordinate indicated below. The coordinates indicated are (-1, 25).The tangent to a curve at a point is given by the first derivative of the curve at the point.

We need to differentiate the given function to get the first derivative of f(x). We get:f(x) = (-4x² + 4x + 3) (-x² - 4)f'(x) = [-8x + 4] (-x² - 4) + (-4x² + 4x + 3) [-2x]f'(x) = 12x³ - 28x² - 16x + 12f'(-1) = 12(-1)³ - 28(-1)² - 16(-1) + 12 = 32The slope of the tangent at the point (-1, 25) is 32.Using the point-slope equation of a line, we get the equation of the tangent line to the graph of f(x) as follows:y - y₁ = m(x - x₁)Here, m = 32, x₁ = -1 and y₁ = 25.Substituting the values, we get:y - 25 = 32(x + 1)y - 25 = 32x + 32y = 32x + 57

Therefore, the equation of the tangent line to the graph of f(x) at the point (-1, 25) is y = 32x + 57.

To know more about equation of the tangent line Visit:

https://brainly.com/question/6617153

#SPJ11

Other Questions
Let A = [x 9][y 2]Find the values of x and y for which A = A. x = __y = __ Background:Your company has just acquired a new subsidiary that makes two products, Clodhopper and Clodbuster. The Clodhopper is intended for the general consumer market and its demand is higher and more seasonal than the Clodbuster, which is intended for the commercial user market. Both products share a number of component parts and subassemblies, primarily differing in their engine and tiller subassemblies. All of the managers at the acquired company quit the day your company took possession. You and your team have been assigned to run this acquisition. Since all the managers quit, the only planning information you have is historical data.Assignment:Your task is to create material requirements plans for items C, E, H, K, P, and L. Use the MRP form supplied. Fill in all appropriate data (lot size, ordering rule, etc). Do not make any changes to the format/layout of the spreadsheet. This is one place in industry where creativity in presenting data is not appreciated since many of the cells in the forms are normally linked, and changes can lead to significant errors in the results. Use the comment function to show the formula for "projected on-hand inventory." Make sure to include your names on the first page.Part 2 Data:The Assn 2 Student" spreadsheet contains five different worksheets: MPS approved plans for Clodhopper and Clodbuster for the first three months of 2022. Bill of materials data for ClodHopper and ClodBuster, including quantities required per product and assembly times/lead times Inventory records and lot-sizing rules for all subassemblies and component parts Scheduled receipts (production commitments and purchase orders that have been scheduled by previous MPS and MRP plans for delivery in 2022. MRP form to use for the items requested aboveAdditional information: Remember that the FOQ lot-sizing rule allows multiple orders. That is, if your plan requires 450 units in a given period, current inventory is 75 units, and the FOQ order size is 100 units, then you would request 400 units (four orders) from the vendor or assembly production line. For items purchased using the POQ lot-sizing rule, consider that the time between orders begins at the start of the plans. That is, if the POQ period is 3, you can only have planned receipts (or scheduled receipts) in periods 1, 4, 7, and so on. Unlike the other ordering schemes where you first determine when a new order must be received and then schedule the order earlier according to the lead time, for POQ you look at future needs that must be satisfied until another POQ order can be placed. Do not schedule MRP planned orders in the line for scheduled receipt items. Scheduled receipts have already been started or ordered on previous MRP plans for delivery in 2022 and you are not allowed to make any additional entries on this line. The values on this line are to be added to the projected on-hand inventory when each scheduled receipt arrives. Do not reflect the scheduled receipts into planned order releases. Some parts and subassemblies have desired safety stock levels.Be sure to order enough or early enough to ensure these levels are maintained, limited only by any lead-time constraints. When there is no safety stock, maintain the projected on-hand greater than or equal to zero. Use the power of Excel as much as possible for filling out your MRP forms. Doing so will save you considerable time in completing your assignment and is representative of how these processes are done in industry. Use the comment function to show the formula for "projected on-hand inventory."Hint: Entering new order quantities (planned receipts) when needed is best done manually whenever the on-hand inventory values indicate that you would not have enough unless an order is placed. You are offered these options on a coin toss: $8 whatever the outcome; or $10 if heads and +$200 if tails. You choose the $8 option. What strategy are you employing O Random strategy O Dominant strategy Maximin strategy O Copycat strategy QUESTION 78 Which political philosophy would most likely lead to the most amount of redistribution from rich to poor? O Libertarianism O Liberalism O Utilitarianism O Capitalism QUESTION 79 "The aggregate demand for good X is Q-20-0 SP 2 (P-squared). If the price rises from P=$4 to P=56, what is the change in consumer surplus?" O $42 O $62 O $82 O $102 QUESTION 80 The government wants to stop smoking Which of the following will reduce smoking and the revenues of cigarette companies? O A reduction in the supply of cigarettes that can be sold OA campaign that shifts the demand for cigarettes in at any price O Acampaign that moves the demand outward O A price celling on cigaretes what is the benefit of biodiversity to humans? variety of food sourcesproduction of oxygen for aerobic respiration to make atpenergysource of drugsall answers are correct. recent data shows that the lives and livelihoods of 345 million people around the world are in immediate danger of acute food insecurity. The execution phase of the CFA Institute's investment management process uses details of optimal asset allocation and security selection.TrueFalse2.The CFA Institute divides the process of portfolio management into three main elements, which are planning, execution, and results.TrueFalse Please asap i have only 30 minutes. answer in your own termsSUBJECT: Legal Environment of Business USAFind a legal case where a judge issued a summary judgment andsummarize the case. question 19in this list of numbers, what is the median? 97, 96, 95, 93, 93, 90, 87, 86, 84, 78, 75, 74, 70, 68, 65.9383.48680 Briefly explain the difficulties of using target costing inservice industry Distinguish between" Systems/Systemic/Holistic thinking and "Un-systemic/Traditional thinking(20 marks ) A large grocery supplier has decided to tackle the amount of perfectly good food going to waste across their supply chain, in the hope that they can reroute goods to local community groups that would benefit from such action. You have been employed as a supply chain expert to help achieve this. What are the first 3 steps you would take? Use the method of variation of parameters to find a particular solution to the following differential equation. 5x y" - 10y + 25y 81+x -1/2*e^(5*x)*In(x^2+81)+(x*e^(5*x Enter your answer as a symbolic function of x, as in these For a given cylindrical tank, the radius is 2 m and the height is 7 m. The tank is filled to a depth of 6 m. How much work is required to pump all of the water over the top edge of the tank? Acceleration due to gravity is 9.8 m/sec and the density of water is 1000 kg/m. Round your answer to the nearest kilojoule. in terms of body position, the opposite of cranial is:group of answer choicesvisceral.caudal.superior.lateral.medial. Two components of a minicomputer have the following joint pdf for their useful lifetimes X and Y: ; x >0 ; y > 0 0 ; elsewhere -y1+ fx x(x, y) = { ye*(1+x) (a) Compute the marginal pdf of Y. Report a complete pdf. (b) Are the two variables independent based on probability? Explain. describe how gerrymandering can be used to influence the election of representatives to congress. find the period of revolution of a satellite moving in a circular orbit around the earth at a height of 3.6 x 10^6 m above the earths surface. Assume the earth is a uniform sphere of radius 6.4 x 10^6 m. The earths mass 6 x 10^24 kg and G= 6.7 x 10^-11 nm^2 a fireman of mass 80 kgkg slides down a pole. for the steps and strategies involved in solving a similar problem, you may view a_____. If You Have A Stack That Contains 3000 Integers, And You Need To See If It Contains The Number 5678, What Is The Worst Case Scenario In Terms Of The Number Of Operations You Would Have To Do? You Can Use A Second Stack. Count Every Peek, Push, Pop And Comparison, And Assume You Need To Ensure That The Integers In The Collection Maintain Their Order In TheIf you have a stack that contains 3000 integers, and you need to see if it contains the number 5678, what is the worst case scenario in terms of the number of operations you would have to do? You can use a second stack. Count every peek, push, pop and comparison, and assume you need to ensure that the integers in the collection maintain their order in the original stack. Find an orthogonal or unitary diagonalizing matrix for each of the following:a. [ 1 3+i] b. [1 1 1][3-i 4] [1 1 1][1 1 1]